Page 1 of 1

PT37, LR1, Q19

Posted: Tue Oct 12, 2010 10:01 am
by gambelda
I am completely confused by this question and have been struggling with Sufficient/Necessary assumptions lately.

So I realize that this is a necessary assumption question. I want to know why answer choice (A) is incorrect. If the individual counseling sessions played a MAJOR role in helping people to quit, then that would completely undermine the physician's argument. So wouldn't answer choice (A) be a defender answer?

On a sidenote, while typing this question up I came to realize that requiring the sessions to "not help at all" is not the only necessary option. They could help "just a little". Does this observation play into the answer?

Re: PT37, LR1, Q19

Posted: Tue Oct 12, 2010 11:04 am
by Manhattan LSAT Noah
This is a tricky one. I don't find the categories of answers (i.e. "defender") necessary (but perhaps they're sufficient :D ), but I'll explain it in more general terms:

The conclusion of this argument is that the yoga is a powerful tool. Why? Because a group that used it, along with counseling, reduced their smoking (to paraphrase) as much as a different group that did self-help groups as well as the counseling.

We're looking for a necessary assumption, so we should look for gaps. Right off the bat, what about that counseling? Couldn't that be the reason both groups were able to reduce their smoking? We need the counseling to not be the reason. So, (A) looks very tempting. However, (A) is slightly different than what I've just stated. (A) has the counseling not helping. I want to know that the counseling is not the reason both groups reduced their smoking. So, (A) is too weak here.

(C), on the other hand, works in a surprising manner. If both groups were able to reduce their smoking, and we need the counseling to not be the reason, how did the non-yoga folks reduce their smoking? It must have been the self-help group. If it wasn't (negating the answer), then the non-yoga people must have been helped by the counseling, which would undermine the argument.

(B) is out of scope - we're not interested in whether the treatment fits into folks' schedules.
(D) is tempting, but the argument is only about smoking, not about overall health.
(E) is similarly out of scope - we aren't concerned with whether hatha yoga is the only yoga that works.

Does that help?

Re: PT37, LR1, Q19

Posted: Tue Oct 12, 2010 11:39 am
by gambelda
So I see how you negated an answer choice and attacked the argument's conclusion. I understand your reasoning in that if you can draw an inference from the non-yoga group then it affects the group that did preform yoga. However, I am still very confused as to why answer choice (C) is correct.

if Traditional self-help groups are powerful, that does not prevent individual counseling from being powerful too. So let's assume that both are equally powerful...why then does confirming that the traditional self-help groups are powerful, indicate that hatha yoga is also powerful?

Maybe I'm just confusing myself further...

Your explanations certainly helps me see why (A) is wrong though. Just because individual counseling did "not help" does not make yoga powerful. There could be another factor that affected their success. Just because one thing did not help, does not mean that the other did help greatly.

Re: PT37, LR1, Q19

Posted: Tue Oct 12, 2010 12:03 pm
by Manhattan LSAT Noah
gambelda wrote:
if Traditional self-help groups are powerful, that does not prevent individual counseling from being powerful too. So let's assume that both are equally powerful...why then does confirming that the traditional self-help groups are powerful, indicate that hatha yoga is also powerful?
And that's why this isn't a sufficient assumption. While the self-help being powerful doesn't prove that the counseling was not also powerful, we need that self-help to be powerful, because if it isn't it would have to be the counseling that did everything. Does that clarify it?

Re: PT37, LR1, Q19

Posted: Tue Oct 12, 2010 2:45 pm
by gambelda
yes, thank you for the help. Clearly, I can improve on necessary and sufficient assumption questions. 5 of the 11 total LR questions I missed on my last exam were assumptions.

As soon as I do great on my RC my LR goes down the drain :roll: